A CR -- less articles

This topic has expert replies
Master | Next Rank: 500 Posts
Posts: 121
Joined: Mon Apr 04, 2016 6:31 pm
Thanked: 1 times

A CR -- less articles

by zoe » Sun Jun 12, 2016 5:48 pm
Dear friends,
Please help explain A,B, D,
thanks in advance.

Journalist: In physics journals, the number of articles reporting the results of experiments involving particle accelerators was lower last year than it had been in previous years. Several of the particle accelerators at major research institutions were out of service the year before last for repairs, so it is likely that the low number of articles was due to the decline in availability of particle accelerators.
Which of the following, if true, most seriously undermines the journalist's argument?
(A) Every article based on experiments with particle accelerators that was submitted for publication last year actually was published.
(B) The average time scientists must wait for access to a particle accelerator has declined over the last several years.
(C) The number of physics journals was the same last year as in previous years.
(D) Particle accelerators can be used for more than one group of experiments in any given year.
(E) Recent changes in the editorial policies of several physics journals have decreased the likelihood that articles concerning particle-accelerator research will be accepted for publication


IMO:
(A) says one reason that cause low number of articles, so I think (A) can weaken the argument. while, OG says (A) eliminates the 3rd cause to rend support. I cannot get the OG's idea,
(B) says the waiting time decline, seems more chance to get accelerators, so I think it weaken the argument.
(D) says the accelerators can be used multi-experiments, that weaken the "decline availability", so I think (D) can weaken the argument as well.

please clarify my errors.

waiting for your reply.

thanks so much.
have a nice day.

>_~

GMAT/MBA Expert

User avatar
GMAT Instructor
Posts: 2095
Joined: Tue Dec 04, 2012 3:22 pm
Thanked: 1443 times
Followed by:247 members

by ceilidh.erickson » Tue Jun 14, 2016 8:47 am
This question is #69 in OG2016.

If we're asked to UNDERMINE (weaken) an argument, we first must find the logical flaw between the premises and conclusion.

Premises:
- the number of articles was lower last year
- several particle accelerators were out of service

Conclusion:
- fewer articles = due to decline in activity of particle accelerators

Logical Gaps:
- does the number of articles directly correlate to the number of experiments? Or might there be some other factor influencing what these journals want to publish? Quality of experiments, general interest, the state of the publishing industry, etc.
- does the number of accelerators directly influence the number of experiments conducted? Or could there have been more total experiments performed, even if a few accelerators were out of service?

We need a new piece of information that disrupts the assumption that number of accelerators --> number of experiments --> number of articles.

(A) Every article based on experiments with particle accelerators that was submitted for publication last year actually was published.
This would actually strengthen the argument, by giving us a direct connection between number of articles and number of experiments. That's the opposite of what we're looking for.
(A) says one reason that cause low number of articles, so I think (A) can weaken the argument
To your question - this isn't giving us a different explanation than the journalist's, because it's an extension of the same causation chain: number of accelerators --> number of experiments --> number of articles

(B) The average time scientists must wait for access to a particle accelerator has declined over the last several years.
It's unclear how this would affect the argument. We're looking for the relationship between low articles and accelerators. Whether scientists had to wait or not would not necessarily affect whether articles were published about it.
(B) says the waiting time decline, seems more chance to get accelerators, so I think it weaken the argument.
You're making too many assumptions here. Just because the waiting time declined, that doesn't belie the given premise that there were several accelerators unavailable. We have to treat our premises as TRUE. We don't weaken by contradicting the premise, but by revealing the flawed connection between premise and conclusion.

(C) The number of physics journals was the same last year as in previous years.
We have no direct reason to believe that the number of journals would directly affect the number of articles published about these accelerators. And since this is a "no change" answer, it wouldn't weaken - if anything, it would strengthen.

(D) Particle accelerators can be used for more than one group of experiments in any given year.
(D) says the accelerators can be used multi-experiments, that weaken the "decline availability", so I think (D) can weaken the argument as well.
No, this does not weaken "decline in availability." Even if we can use them for multiple experiments, out-of-service accelerators could still mean fewer experiments performed overall, leading to fewer articles. Or it could mean more experiments. There is just no direct tie to what this means about the articles.

(E) Recent changes in the editorial policies of several physics journals have decreased the likelihood that articles concerning particle-accelerator research will be accepted for publication
Bingo! This gives us an external factor that completely disrupts the connection between number of articles = number of experiments. If the journals are less likely to publish these articles, it doesn't matter whether there was a decline in activity or not.

The correct answer is E.
Ceilidh Erickson
EdM in Mind, Brain, and Education
Harvard Graduate School of Education

GMAT/MBA Expert

User avatar
GMAT Instructor
Posts: 2095
Joined: Tue Dec 04, 2012 3:22 pm
Thanked: 1443 times
Followed by:247 members

by ceilidh.erickson » Tue Jun 14, 2016 8:51 am
Ceilidh Erickson
EdM in Mind, Brain, and Education
Harvard Graduate School of Education

Master | Next Rank: 500 Posts
Posts: 199
Joined: Sat Apr 26, 2014 10:53 am
Thanked: 16 times
Followed by:4 members
GMAT Score:780

by 800_or_bust » Tue Jun 14, 2016 10:03 am
zoe wrote:Dear friends,
Please help explain A,B, D,
thanks in advance.

Journalist: In physics journals, the number of articles reporting the results of experiments involving particle accelerators was lower last year than it had been in previous years. Several of the particle accelerators at major research institutions were out of service the year before last for repairs, so it is likely that the low number of articles was due to the decline in availability of particle accelerators.
Which of the following, if true, most seriously undermines the journalist's argument?
(A) Every article based on experiments with particle accelerators that was submitted for publication last year actually was published.
(B) The average time scientists must wait for access to a particle accelerator has declined over the last several years.
(C) The number of physics journals was the same last year as in previous years.
(D) Particle accelerators can be used for more than one group of experiments in any given year.
(E) Recent changes in the editorial policies of several physics journals have decreased the likelihood that articles concerning particle-accelerator research will be accepted for publication


IMO:
(A) says one reason that cause low number of articles, so I think (A) can weaken the argument. while, OG says (A) eliminates the 3rd cause to rend support. I cannot get the OG's idea,
(B) says the waiting time decline, seems more chance to get accelerators, so I think it weaken the argument.
(D) says the accelerators can be used multi-experiments, that weaken the "decline availability", so I think (D) can weaken the argument as well.

please clarify my errors.

waiting for your reply.

thanks so much.
have a nice day.

>_~
The correct answer is actually E. The remaining answer choices are irrelevant to the argument presented in the passage.
800 or bust!

Master | Next Rank: 500 Posts
Posts: 121
Joined: Mon Apr 04, 2016 6:31 pm
Thanked: 1 times

by zoe » Thu Jun 16, 2016 6:54 pm
ceilidh.erickson wrote:This question is #69 in OG2016.

If we're asked to UNDERMINE (weaken) an argument, we first must find the logical flaw between the premises and conclusion.

Premises:
- the number of articles was lower last year
- several particle accelerators were out of service

Conclusion:
- fewer articles = due to decline in activity of particle accelerators

Logical Gaps:
- does the number of articles directly correlate to the number of experiments? Or might there be some other factor influencing what these journals want to publish? Quality of experiments, general interest, the state of the publishing industry, etc.
- does the number of accelerators directly influence the number of experiments conducted? Or could there have been more total experiments performed, even if a few accelerators were out of service?

We need a new piece of information that disrupts the assumption that number of accelerators --> number of experiments --> number of articles.

(A) Every article based on experiments with particle accelerators that was submitted for publication last year actually was published.
This would actually strengthen the argument, by giving us a direct connection between number of articles and number of experiments. That's the opposite of what we're looking for.
(A) says one reason that cause low number of articles, so I think (A) can weaken the argument
To your question - this isn't giving us a different explanation than the journalist's, because it's an extension of the same causation chain: number of accelerators --> number of experiments --> number of articles
thanks ceilidh, thanks for your excellent explanation.

for A, i totally agree with you, but i am afraid i need more help because i haven't getten the idea why A is incorrect, i thought again and again after reading your explanation on A, would you please point out my fault...
(A) Every article based on experiments with particle accelerators that was submitted for publication last year actually was published
in a short , every article that was submitted acutally was published...
I think A means that articles cann't be published because they was published before, that the 3rd reason cause low number of articles...

if we want to weaken the conclusion, we can find/show 3rd cause which can cause the effect..
in this question, the cause = low avialibility of accelerator, effect = low nomber of articles,
you can realize, A shows a 3rd cause that cause the low number of articles. that's why i think A weaken the argument.
while both you and OA think A strengthen the argument..
i don't know what is my fault

please help again..

thanks a lot
have a nice day
>_~

GMAT/MBA Expert

User avatar
GMAT Instructor
Posts: 2095
Joined: Tue Dec 04, 2012 3:22 pm
Thanked: 1443 times
Followed by:247 members

by ceilidh.erickson » Fri Jun 17, 2016 6:58 am
for A, i totally agree with you, but i am afraid i need more help because i haven't getten the idea why A is incorrect, i thought again and again after reading your explanation on A, would you please point out my fault...
(A) Every article based on experiments with particle accelerators that was submitted for publication last year actually was published
in a short , every article that was submitted acutally was published...
I think A means that articles cann't be published because they was published before, that the 3rd reason cause low number of articles...

if we want to weaken the conclusion, we can find/show 3rd cause which can cause the effect..
in this question, the cause = low avialibility of accelerator, effect = low nomber of articles,
you can realize, A shows a 3rd cause that cause the low number of articles. that's why i think A weaken the argument.
while both you and OA think A strengthen the argument..
i don't know what is my fault
The conclusion of the argument is "it is likely that the low number of articles was due to the decline in availability of particle accelerators." But the articles in question are "articles reporting the results of experiments." The assumption that we're being asked to make is that lower availability of accelerators last year --> lead to fewer experiments performed last year --> lead to fewer articles published last year.

I'm not sure exactly what you mean by
I think A means that articles cann't be published because they was published before, that the 3rd reason cause low number of articles...
The entire scope of the argument is what happened last year. If last year every accelerator yielded the same number of experiments performed, and every experiment performed yielded one article written, and every article written was published, this chain of logic would hold. I think you're maybe interpreting A as a comparison to some other time period?
Ceilidh Erickson
EdM in Mind, Brain, and Education
Harvard Graduate School of Education

Master | Next Rank: 500 Posts
Posts: 121
Joined: Mon Apr 04, 2016 6:31 pm
Thanked: 1 times

by zoe » Fri Jun 17, 2016 9:01 pm
ceilidh.erickson wrote:
I'm not sure exactly what you mean by
I think A means that articles cann't be published because they was published before, that the 3rd reason cause low number of articles...
ceilidh.erickson wrote: The entire scope of the argument is what happened last year. If last year every accelerator yielded the same number of experiments performed, and every experiment performed yielded one article written, and every article written was published, this chain of logic would hold. I think you're maybe interpreting A as a comparison to some other time period?

thanks so much again Ceilidh,

I think I know why you didn't catch what I say.

obviously you think of this question as a chain issue, I did not realize that at beginning, thank you for your mention. also, I read all links you told me, all these links are chain flaws in the questions.

I think of this question as a cause - effect question, because here is a obviously indicator "due to". so the cause is decine availability, the effect is low number of articles...

i have read both BIBLE and MANHATTAN, one difference between these two is that cause effect question is one type in BIBLE, while there is no CE (cause effect)type in MANHATTAN.

maybe that's why you did not catch what i say.

I like your excellect explanation...it helped a lot.

i just want to know why A is incorrect if consider it a CE question, because i am weak on CE questions, always can't distinguish Weaken or strengthen choices..
so bad.....and hope to overcome it.

thank you for your excellent explanation and your patience.

it will be great appreciate if you can point out my flaw if consider it CE question.

have a nice day
>_~